Đến nội dung

NTA1907 nội dung

Có 1000 mục bởi NTA1907 (Tìm giới hạn từ 28-04-2020)



Sắp theo                Sắp xếp  

#655348 Topic về phương trình và hệ phương trình

Đã gửi bởi NTA1907 on 24-09-2016 - 13:00 trong Phương trình - hệ phương trình - bất phương trình

Bài 541: $\left\{\begin{matrix} &xy+\sqrt{2(x^{4}+y^{4})}=1 \\ &x^{2009}y^{2013}+x^{2013}y^{2009}=\dfrac{2}{3^{2011}} \end{matrix}\right.$




#655226 $\sqrt{2x-2}+\sqrt[3]{x-2}=\frac...

Đã gửi bởi NTA1907 on 23-09-2016 - 12:58 trong Phương trình, hệ phương trình và bất phương trình

Giải phương trình 

$\sqrt{2x-2}+\sqrt[3]{x-2}=\frac{9-x}{\sqrt[3]{8x-16}}$

Điều kiện: $x\geq 1, x\neq 2$

+) Với $1\leq x< 2\Rightarrow VT\geq -1, VP\leq -4\Rightarrow$ Phương tình vô nghiệm

+) Với $x> 2$. Phương trình đã cho tương đương:

$\left ( \sqrt{2x-2}-2 \right )+\left ( \sqrt[3]{x-2}-1 \right )+\left ( \frac{x-9}{\sqrt[3]{8x-16}}+3 \right )=0$

$\Leftrightarrow \frac{2(x-3)}{\sqrt{2x-2}+2}+\frac{x-3}{\sqrt[3]{(x-2)^{2}}+\sqrt[3]{x-2}+1}+\frac{(x-3)(x^{2}-24x+387)}{\sqrt[3]{8x-16}\left [ (x-9)^{2}-3(x-9)\sqrt[3]{8x-16}+9\sqrt[3]{(8x-16)^{2}} \right ]}=0$

$\Leftrightarrow x=3$(thoả mãn) hoặc $\frac{2}{\sqrt{2x-2}+2}+\frac{1}{\sqrt[3]{(x-2)^{2}}+\sqrt[3]{x-2}+1}+\frac{x^{2}-24x+387}{\sqrt[3]{8x-16}\left [ (x-9)^{2}-3(x-9)\sqrt[3]{8x-16}+9\sqrt[3]{(8x-16)^{2}} \right ]}=0(*)$

(*) vô nghiệm $\vee x> 2$

Vậy phương trình có nghiệm duy nhất $x=3.$




#655198 $2\sqrt{\left ( 2-x \right )\left ( 5-x \right...

Đã gửi bởi NTA1907 on 22-09-2016 - 22:18 trong Phương trình, hệ phương trình và bất phương trình

Giải phương trình

$2\sqrt{\left ( 2-x \right )\left ( 5-x \right )}=x+\sqrt{\left ( 2-x \right )\left ( 10-x \right )}$

Điều kiện: $x\leq 2$ hoặc $x\geq 10$

Phương trình đã cho tương đương với:

$2\left [ \sqrt{x^{2}-7x+10}-(x+1) \right ]=\sqrt{x^{2}-12x+20}-(x+2)$

$\Leftrightarrow \frac{-18(x-1)}{\sqrt{x^{2}-7x+10}+x+1}=\frac{-16(x-1)}{\sqrt{x^{2}-12x+20}+x+2}$

$\Leftrightarrow x=1$(thoả mãn) hoặc $\frac{9}{\sqrt{x^{2}-7x+10}+x+1}=\frac{8}{\sqrt{x^{2}-12x+20}+x+2}(*)$

$(*)\Leftrightarrow 8\sqrt{x^{2}-7x+10}=x+10+9\sqrt{x^{2}-12x+20}$

Kết hợp với phương trình đầu ta được:

$5\sqrt{x^{2}-12x+20}=3x-10\Rightarrow x=\frac{15+5\sqrt{5}}{2}$




#655182 $(x-y)(y-z)(z-x)(x+y+z)=\dfrac{2\sqrt{3}}...

Đã gửi bởi NTA1907 on 22-09-2016 - 21:44 trong Phương trình - hệ phương trình - bất phương trình

Giải hệ phương trình:

$\left\{\begin{matrix} &\sqrt{x-x^{2}}+\sqrt{y-y^{2}}+\sqrt{z-z^{2}}=\sqrt{\dfrac{\sqrt{3}-1}{3}} \\ &(x-y)(y-z)(z-x)(x+y+z)=\dfrac{2\sqrt{3}}{9} \end{matrix}\right.$

 




#655173 GHPT: $(2x+1)\sqrt{4x-5}+(2y+1)\sqrt{4y-5}...

Đã gửi bởi NTA1907 on 22-09-2016 - 21:22 trong Phương trình - hệ phương trình - bất phương trình

Giải hệ phương trình: 

$\left\{\begin{matrix}(2x+1)\sqrt{4x-5}+(2y+1)\sqrt{4y-5}=2\sqrt{(2x+1)(2y+1)} \\ 5(x+y)=4xy+6 \end{matrix}\right.$

Điều kiện: $x\geq \frac{5}{4}, y\geq \frac{5}{4}$

Từ phương trình 2 ta có: $(4x-5)(4y-5)=1$

Áp dụng bất đẳng thức AM-GM ta có:

$VT_{(1)}\geq 2\sqrt{(2x+1)(2y+1)\sqrt{(4x-5)(4y-5)}}=2\sqrt{(2x+1)(2y+1)}=VP_{(1)}$

Dấu "=" xảy ra$\Leftrightarrow x=y=\frac{3}{2}$(thoả mãn)




#654865 $x^{3}+(y^{2}-2y-2)x+18y=3x^{2}+5y^{2...

Đã gửi bởi NTA1907 on 20-09-2016 - 13:48 trong Phương trình - hệ phương trình - bất phương trình

Giải hệ phương trình:

$\left\{\begin{matrix} &x^{3}+(y^{2}-2y-2)x+18y=3x^{2}+5y^{2}+12 \\ &y^{3}+(x^{2}+2x+4)y=4x^{2}+6y^{2}-6 \end{matrix}\right.$




#654863 giải pt: $sin^3x+cos^3x=2-sin^4x$

Đã gửi bởi NTA1907 on 20-09-2016 - 13:27 trong Phương trình, Hệ phương trình Lượng giác

Giải pt: $sin^3x+cos^3x=2-sin^4x$

Ta có:

$VT=sin^{3}x+cos^{3}x\leq sin^{2}x+cos^{2}x=1$

$VP=2-sin^{4}x\geq 2-1=1$

$\Rightarrow VT=VP=1$

Dấu "=" xảy ra$\Leftrightarrow \left\{\begin{matrix} &sinx=1 \\ &cosx=0 \end{matrix}\right.$

$\Leftrightarrow x=\frac{\pi }{2}+k\pi$




#654861 $(x+1)\sqrt{x+2}+(x+6)\sqrt{x+7}\geq...

Đã gửi bởi NTA1907 on 20-09-2016 - 13:04 trong Phương trình - hệ phương trình - bất phương trình

Giải bpt:

$(x+1)\sqrt{x+2}+(x+6)\sqrt{x+7}\geq x^{2}+7x+12$

 

 

Lời giải.

Điều kiện xác định: $x\geq -2$.

Ta có:

$$\left ( x+1 \right )\sqrt{x+2}+\left ( x+6 \right )\sqrt{x+7}\geq x^{2}+7x+12$$

$$\Leftrightarrow \left ( x+1 \right )\left ( \sqrt{x+2}-2 \right )+\left ( x+6 \right )\left ( \sqrt{x+7}-3 \right )\geq x^{2}+2x-8$$
$$\Leftrightarrow \dfrac{\left ( x+1 \right )\left ( x-2 \right )}{\sqrt{x+2}+2}+\dfrac{\left ( x+6 \right )\left ( x-2 \right )}{\sqrt{x+7}+3}\geq \left ( x-2 \right )\left ( x+4 \right )$$
$$\Leftrightarrow \left ( x-2 \right )\left ( \dfrac{x+1}{\sqrt{x+2}+2}+\dfrac{x+6}{\sqrt{x+7}+3}-x-4 \right )\geq 0$$
Mặt khác ta có:
\begin{align*} \dfrac{x+1}{\sqrt{x+2}+2}+\dfrac{x+6}{\sqrt{x+7}+3} &=\dfrac{x+2}{\sqrt{x+2}+2}+\dfrac{x+6}{\sqrt{x+7}+3}-\dfrac{1}{\sqrt{x+2}+2} \\ &=\left ( x+2 \right )\left ( \dfrac{1}{\sqrt{x+2}+2}-\dfrac{1}{2} \right )+\left ( x+6 \right )\left ( \dfrac{1}{\sqrt{x+7}+3}-\dfrac{1}{3} \right )-\dfrac{1}{\sqrt{x+2}+2}<0 \end{align*}
$$\Rightarrow \dfrac{x+1}{\sqrt{x+2}+2}+\dfrac{x+6}{\sqrt{x+7}+3}<\dfrac{x+2}{2}+\dfrac{x+6}{3}<x+4$$
$$\Rightarrow \dfrac{x+1}{\sqrt{x+2}+2}+\dfrac{x+6}{\sqrt{x+7}+3}-x-4<0$$
Do đó bất phương trình tương đương:
$$x-2\leq 0$$
$$\Leftrightarrow x\geq 2$$
Kết hợp điều kiện ta được $-2\leq 2\leq 2$.

 

Cách liên hợp không cần đánh giá...

 

Điều kiện: $x\geq -2$

Phương trình đã cho tương đương:

$3x^{2}+21x+36-3(x+1)\sqrt{x+2}-3(x+6)\sqrt{x+7}\leq 0$

$\Leftrightarrow x^{2}+3x-10+(x+1)\left ( x+4-3\sqrt{x+2} \right )+(x+6)\sqrt{x+7}\left ( \sqrt{x+7}-3 \right )\leq 0$

$\Leftrightarrow (x-2)(x+5)+\frac{(x+1)^{2}(x-2)}{x+4+3\sqrt{x+2}}+\frac{(x+6)\sqrt{x+7}(x-2)}{\sqrt{x+7}+3}\leq 0$

$\Leftrightarrow (x-2)\left ( x+5+\frac{(x+1)^{2}}{x+4+3\sqrt{x+2}}+\frac{(x+6)\sqrt{x+7}}{\sqrt{x+7}+3} \right )\leq 0$

Vì phần trong ngoặc luôn dương với mọi $x\geq -2$ nên $x\leq 2$

$\Rightarrow -2\leq x\leq 2$




#654858 Topic về phương trình và hệ phương trình

Đã gửi bởi NTA1907 on 20-09-2016 - 12:49 trong Phương trình - hệ phương trình - bất phương trình

Thảo luận hướng đi bài 534: 

Điều kiện: $y\geq \frac{3}{2};x\geq 2$.

Hệ phương trình viết lại: $\left\{\begin{matrix}(\sqrt{4xy+33}-\sqrt{2y-3})(\frac{3(\sqrt{4xy+33}+\sqrt{2y-3})}{y}-1)=0 \\ (x-y-2)(\frac{8y}{\sqrt{xy-2y}+y}+y-x-2)=0 \end{matrix}\right.$

 

 

P/S: Cho hỏi NTA1907. Bài này có nghiệm ko ? Để mình biết hướng đi của bài. 

Vì nếu hai cái loằng ngoằng kia vô nghiệm thì ta được hệ: $\left\{\begin{matrix}4xy+33=2y-3 \\ x-y-2=0 \end{matrix}\right.(VN)$

Hệ này cũng vô nghiệm. 

Bài này vô nghiệm  :) 

 

P/s: Hình như mọi người đánh sai STT bài rồi 

Bài 538: $\left\{\begin{matrix} &y^{7}+1=(x+1)(x^{2}+1)(x^{4}+1) \\ &x^{7}+1=(y+1)(y^{2}+1)(y^{4}+1) \end{matrix}\right.$




#654638 Topic về phương trình và hệ phương trình

Đã gửi bởi NTA1907 on 18-09-2016 - 12:43 trong Phương trình - hệ phương trình - bất phương trình

Bài 534: $\left\{\begin{matrix} &12x+\dfrac{108}{y}-6=\sqrt{4xy+33}-\sqrt{2y-3} \\ &8\sqrt{xy-2y}-8y+4=(x-y)^{2} \end{matrix}\right.$




#654618 Topic về phương trình và hệ phương trình

Đã gửi bởi NTA1907 on 18-09-2016 - 10:25 trong Phương trình - hệ phương trình - bất phương trình

Bài này đã sửa đề rồi? 

Từ PT thứ 2, ta thu được $x=y=0$ (!!!???)

 

P/S: Bài 531 của NTA1907 hình như sửa lại vẫn sai vậy.

Chắc đề ban đầu đúng 




#654441 Topic về phương trình và hệ phương trình

Đã gửi bởi NTA1907 on 16-09-2016 - 23:06 trong Phương trình - hệ phương trình - bất phương trình

Lời giải bài 531:

Từ phương trình đầu, ta có: $1-xy=\sqrt{(xy)^4-(x^2+y^2)+1}\leq \sqrt{(xy)^4-2xy+1}$.

Bình phương hai vế, ta được: $xy\in (-\infty ;-1]\cup [1;+\infty)$. $(*)$

Từ phương trình hai, ta có điều kiện $-1\leq x,y\leq 1$.

Do đó: $-1\leq xy\leq 1$. $(**)$.

Từ $(*),(**)$, ta được: $xy=\pm 1$.

Dấu bằng xảy ra ở $(*)$ khi $x=y$.

Suy ra $x=y=\pm 1$.

 

P/S: Không biết đề đúng không ? NTA1907.

Đã sửa...Bài này có nghiệm $(x,y)=(0;0)$




#654375 $\sum \frac{a^{2}-b^{2}}{...

Đã gửi bởi NTA1907 on 16-09-2016 - 14:16 trong Bất đẳng thức - Cực trị

Cho $a,b,c>0$. Chứng minh rằng:

$\frac{a^{2}-b^{2}}{\sqrt{b+c}}+\frac{b^{2}-c^{2}}{\sqrt{c+a}}+\frac{c^{2}-a^{2}}{\sqrt{a+b}}\geq 0$

 




#654373 Topic về phương trình và hệ phương trình

Đã gửi bởi NTA1907 on 16-09-2016 - 13:00 trong Phương trình - hệ phương trình - bất phương trình

 

Bài 529: Giải hệ phương trình:

$$\left\{\begin{matrix} y^{6}+y^{3}+2x^{2}=\sqrt{xy-x^{2}y^{2}} \\ 8xy^{3}+2y^{3}+\dfrac{1}{2}=4x^{4}+3x^{2}+x+2\sqrt{1+\left ( 2x-y \right )^{2}} \end{matrix}\right.$$
----
Từ hai bài toán trên có thể thấy được một hướng chế đề từ các tổng bình phương khá hay :D

 

Em thấy đề phương trình 2 phải là $8xy^{3}+2y^{3}+\dfrac{1}{2}=4x^{4}+3x^{2}+x+2\sqrt{1+\left ( 2x-y \right )^{2}}$ mới phân tích được thành tổng bình phương.

 

Điều kiện: $0\leq xy\leq 1$

Áp dụng bất đẳng thức AM-GM ta có:

$y^{6}+y^{3}+2x^{2}=\sqrt{xy(1-xy)}\leq \frac{xy+1-xy}{2}=\frac{1}{2}$

$\Leftrightarrow 1\geq 2y^{6}+2y^{3}+4x^{2}(*)$

Ta có: $8xy^{3}+2y^{3}+\frac{1}{2}\geq 4x^{4}+3x^{2}+x+2(**)$

Cộng 2 bất đẳng thức (*) và (**) ta được:

$8xy^{3}+2y^{3}+\frac{3}{2}\geq 2y^{6}+2y^{3}+4x^{2}+4x^{4}+3x^{2}+x+2$

$\Leftrightarrow 2y^{6}-8xy^{3}+4x^{4}+7x^{2}+x+\frac{1}{2}\leq 0$

$\Leftrightarrow 2(y^{3}-2x)^{2}+(2x^{2}-\frac{1}{2})^{2}+(x+\frac{1}{2})^{2}\leq 0$

$\Leftrightarrow x=\frac{-1}{2}$ $y=-1$

Vậy hệ đã cho có nghiệm $(x,y)=\left ( \frac{-1}{2};-1 \right )$

 

Bài 530: $\left\{\begin{matrix} &\sqrt{x+3}=y^{3}-6 & \\ &\sqrt{y+2}=z^{3}-25 & \\ &\sqrt{z+1}=x^{3}+1 & \end{matrix}\right.$

Bài 531: $\left\{\begin{matrix} &\sqrt{x^{4}y^{4}-(x^{2}+y^{2})+1}=1-xy \\ &\sqrt{1-x^{2}}+\sqrt{1-y^{2}}=\sqrt{2} \end{matrix}\right.$




#654280 $P=\frac{x^3}{3y+1}+\frac{y^3}...

Đã gửi bởi NTA1907 on 15-09-2016 - 18:43 trong Bất đẳng thức và cực trị

Cho $x,y,z$ là các số thực không âm thỏa mãn $x^3+y^3+z^3=3$. Tìm giá trị nhỏ nhất của biểu thức

                               $P=\frac{x^3}{3y+1}+\frac{y^3}{3z+1}+\frac{z^3}{3x+1}$

Áp dụng AM-GM ta có: $x^{3}+1+1\geq 3x\Leftrightarrow x^{3}+3\geq 3x+1 \Leftrightarrow \frac{z^{3}}{3x+1}\geq \frac{z^{3}}{x^{3}+3}$

$\Rightarrow \sum \frac{z^{3}}{3x+1}\geq \sum \frac{z^{3}}{x^{3}+3}$

Áp dụng bất đẳng thức Schwarz ta có:

$\sum \frac{z^{3}}{x^{3}+3}\geq \sum \frac{z^{6}}{x^{3}z^{3}+3z^{3}}\geq \frac{(\sum z^{3})^{2}}{\sum x^{3}z^{3}+3\sum z^{3}}\geq \frac{(\sum z^{3})^{2}}{\frac{(\sum z^{3})^{2}}{3}+3\sum z^{3}}=\frac{3}{4}$

Dấu "=" xảy ra$\Leftrightarrow x=y=z=1$




#654232 $\sqrt{x}+\sqrt{x^{2}+y+3}=2$

Đã gửi bởi NTA1907 on 14-09-2016 - 22:21 trong Phương trình, hệ phương trình và bất phương trình

Giải hệ phương trình:

$\left\{\begin{matrix} &\sqrt{x}+\sqrt{x^{2}+y+3}=2 \\ &2\sqrt{x+4}+3\sqrt{y+8}=13 \end{matrix}\right.$




#654227 $(x+y)(z+1)=(z+x)(y+1)=(y+z)(x+1)=2a(a+1)$

Đã gửi bởi NTA1907 on 14-09-2016 - 22:05 trong Phương trình - Hệ phương trình - Bất phương trình

Giải hệ phương trình:

$(x+y)(z+1)=(z+x)(y+1)=(y+z)(x+1)=2a(a+1)$




#654225 Topic về phương trình và hệ phương trình

Đã gửi bởi NTA1907 on 14-09-2016 - 21:51 trong Phương trình - hệ phương trình - bất phương trình

Bất đẳng thức Cauchy-Schwarz nếu mình không nhớ không nhầm thì là vầy:

$$\left ( ax+by \right )^{2}\leq \left ( a^{2}+b^{2} \right )\left ( x^{2}+y^{2} \right )$$

Nếu vậy thì sao có đoạn này được nhỉ? Mình không rành bất đẳng thức cho lắm nên có gì sai sót mong bỏ qua...

Đây là lỗi của e khi ghi sai đề. Nhưng nếu bài toán như lúc đầu thì ý tưởng của Lawliet hẳn đã tối ưu nhất chưa

Bài 527: $\left\{\begin{matrix} &x\sqrt{1-y^{2}}+y\sqrt{x^{2}-1}=1 \\ &3x^{2}-xy^{2}+4x=1 \end{matrix}\right.$




#654140 GHPT: $\sqrt{y+2x-1}+\sqrt{1-y}=y+2$

Đã gửi bởi NTA1907 on 14-09-2016 - 13:53 trong Phương trình - hệ phương trình - bất phương trình

Giải hệ phương trình:

$\left\{\begin{matrix}\sqrt{y+2x-1}+\sqrt{1-y}=y+2 \\ x\sqrt{x}=\sqrt{y(x-1)}+\sqrt{x^2-y} \end{matrix}\right.$

Điều kiện: $x\geq 0, y\leq 1, y+2x\geq 1, y(x-1)\geq 0, x^{2}\geq y$

Phương trình 2 tương đương:

$x^{3}=x^{2}+xy-2y+2\sqrt{(xy-y)(x^{2}-y)}$

$\Leftrightarrow x^{3}-x^{2}-xy=2\left ( \sqrt{(xy-y)(x^{2}-y)}-y \right )$

$\Leftrightarrow x^{3}-x^{2}-xy=\frac{2y(x^{3}-x^{2}-xy)}{\sqrt{(xy-y)(x^{2}-y)}+y}$

$\Leftrightarrow (x^{3}-x^{2}-xy)\left ( 1-\frac{2y}{\sqrt{(xy-y)(x^{2}-y)}+y} \right )=0$

$\Leftrightarrow (x^{3}-x^{2}-xy)^{2}=0$

$\Leftrightarrow x=0$ hoặc $x^{2}-x=y$

TH1: $x=0\Rightarrow$ Vô nghiệm

TH2: $x^{2}-x=y$

Thay vào phương trình 1 ta được:

$\sqrt{x^{2}+x-1}+\sqrt{-x^{2}+x+1}=x^{2}-x+2(*)$

Điều kiện: $x^{2}+x-1\geq 0, -x^{2}+x+1\geq 0$

Áp dụng bất đẳng thức AM-GM ta có:

$\sqrt{x^{2}+x-1}\leq \frac{x^{2}+x}{2}$

$\sqrt{-x^{2}+x+1}\leq \frac{-x^{2}+x+2}{2}$

$\Rightarrow VT_{(*)}\leq x+1 \Rightarrow x^{2}-x+2\leq x+1$

$\Leftrightarrow (x-1)^{2}\leq 0\Leftrightarrow x=1$(thoả mãn)

$\Rightarrow y=0$

 

Vậy hệ có nghiệm duy nhất $(x;y)=(1;0)$




#654138 Topic về phương trình và hệ phương trình

Đã gửi bởi NTA1907 on 14-09-2016 - 13:37 trong Phương trình - hệ phương trình - bất phương trình

- Với $2x=y$ phương trình thứ hai của hệ trở thành:

$$\sqrt{12x^{2}+6x+84}=2x+2\sqrt{x+2}+\sqrt{20-2x}$$

Cách đánh giá khác cho phương trình này

 

Điều kiện: $-2\leq x\leq 10$

Áp dụng bất đẳng thức AM-GM ta có:

$VP=2x+\sqrt{4(x+2)}+\frac{1}{4}\sqrt{16(20-2x)}\leq 2x+\frac{1}{2}(6+x)+\frac{1}{8}(36-2x) \Rightarrow \sqrt{12x^{2}+6x+84}\leq 2x+\frac{1}{2}(6+x)+\frac{1}{8}(36-2x)$

$\Leftrightarrow 12x^{2}+6x+84\leq \left ( \frac{9x}{4}+\frac{15}{2} \right )^{2}$

$\Leftrightarrow \frac{111}{16}(x-2)^{2}\leq 0\Leftrightarrow x=2$(thoả mãn)

 

 

 

Bài 523: $\left\{\begin{matrix} &x\sqrt{1-y^{2}}+y\sqrt{1-x^{2}}=1 \\ &3x^{2}-xy^{2}+4x=1 \end{matrix}\right.$

Cách tiếp cận khác so với cách của L Lawliet

 

Điều kiện: $-1\leq x\leq 1, -1\leq y\leq 1$

Áp dụng bất đẳng thức Cauchy-Schwarz ta có:

$VT_{(1)}\leq \sqrt{(x^{2}+1-x^{2})(y^{2}+1-y^{2})}=1$

Dấu "=" xảy ra$\Leftrightarrow \frac{x}{\sqrt{1-x^{2}}}=\frac{\sqrt{1-y^{2}}}{y}\Leftrightarrow x^{2}+y^{2}=1$

Thay $y^{2}=1-x^{2}$ vào phương trình 2 ta được:

$x^{3}+3x^{2}+3x-1=0 \Leftrightarrow (x+1)^{3}=2 \Leftrightarrow x=\sqrt[3]{2}-1$

$\Rightarrow y=\sqrt{1-\left ( \sqrt[3]{2}-1 \right )^{2}}$




#654136 $a^2+b^+c^2\leqslant 8$ CMR $ab+2ac+bc\geq 8$

Đã gửi bởi NTA1907 on 14-09-2016 - 12:44 trong Bất đẳng thức và cực trị

Với a, b, c là các số thực

Có $a^2+b^2+c^2\leqslant 8$ CMR

$ab+2ac+bc\geq -8$

Min bài này phải là -8 nhé...

Ta có:

$8+ab+2ac+bc\geq a^{2}+b^{2}+c^{2}+ab+2ac+bc=(c+a)^{2}+b(c+a)+\frac{b^{2}}{4}+\frac{3b^{2}}{4}=(c+a+\frac{b}{2})^{2}+\frac{3b^{2}}{4}\geq 0$

$\Leftrightarrow ab+2ac+bc\geq -8$

Dấu "=" xảy ra$\Leftrightarrow a=2, b=0, c=-2$ hoặc $a=-2, b=0, c=2$




#654003 $...=\dfrac{1}{x+\dfrac{1}{x+...

Đã gửi bởi NTA1907 on 13-09-2016 - 13:20 trong Phương trình - hệ phương trình - bất phương trình

Giải phương trình:

$1+\dfrac{1}{1+\dfrac{1}{1+\dfrac{1}{\sqrt{1-x}}}}=\dfrac{1}{x+\dfrac{1}{x+\dfrac{1}{\sqrt{x+1}}}}+\frac{2}{3+\sqrt{x}}$

Spoiler

 




#653559 $(x+y)^3+(x+z)^3+3(x+y)(x+z)(y+z)\leq 5(y+z)^3$

Đã gửi bởi NTA1907 on 10-09-2016 - 14:24 trong Bất đẳng thức và cực trị

chứng minh với mọi số thực x,y,z thỏa mãn $x(x+y+z)=3yz$ ta có 

$(x+y)^3+(x+z)^3+3(x+y)(x+z)(y+z)\leq 5(y+z)^3$

đây




#653555 Topic về phương trình và hệ phương trình

Đã gửi bởi NTA1907 on 10-09-2016 - 13:33 trong Phương trình - hệ phương trình - bất phương trình

Giải HPT 

$\left\{\begin{matrix} 2x^{2}y+2(x-2)^{2}=(xy+y+3x-3)y+10\\ y=x^{2}-x+\sqrt{2x-x^{2}}+2 \end{matrix}\right.$

Đánh STT bài 525 vào đi bạn. Lần sau bạn chú ý hơn khi post bài.




#653549 Topic về phương trình và hệ phương trình

Đã gửi bởi NTA1907 on 10-09-2016 - 12:24 trong Phương trình - hệ phương trình - bất phương trình

Lời giải.

- Với $2x=y$ phương trình thứ hai của hệ trở thành:

$$\sqrt{12x^{2}+6x+84}=2x+2\sqrt{x+2}+\sqrt{20-2x}$$

Phương trình này có 1 nghiệm $x=2$ mà chị.  :)